Answers & Explanations

8 downloads 191264 Views 114KB Size Report
Copyright © 2003 Examkrackers, Inc. Passage I (Questions 1-6). 1. C is correct. If the molar amounts are equal, we can use a sample that contains one mole of ...
Answers & Explanations Physical Sciences Questions 1 - 77

65

Passage I (Questions 1-6) 1. C is correct. If the molar amounts are equal, we can use a sample that contains one mole of each metal. The percent by mass of gold of the sample will be 197/(197 + 63.5). That’s about 75%, which is the same as 18/24, or 18 carats.

Passage IX (Questions 13-18) 13. A is correct. Only positive particles will be curved in the direction that will cause them to hit the detector. Negative particles will be curved in the opposite direction. 14. D is correct. As the particle moves through the voltage difference, potential energy is converted to kinetic energy. qV = 1/2 mv2. Solve for v. v = sq rt. of 2Vq/m.

2. A is correct. From the passage, cast iron is stronger and harder to work than regular iron, so it fits the description of an interstitial alloy.

15. C is correct. From the mass-charge ratio equation in the passage, larger mass and smaller charge will make the radius larger.

3. D is correct. Lead has the smallest Young’s modulus on the table, so it is the most ductile, and it has the greatest resistance, so it is the least conductive.

16. D is correct. Decreasing V will decrease the speed of the particle, which will decrease r. Increasing B will increase the centripetal force, which will also decrease r.

4. D is correct. Use the power formula P = I2R. Coulombs per second are amperes, so the current is 5 A. The resistance for copper is 0.02 ohms per meter, so for 1 kilometer, the resistance is 20 ohms. P = (5)2(20) = 500 W.

17. B is correct. Using the right hand rule, the magnetic field will push a positive particle to the right, so the electric field should point towards the left to oppose the effects of the magnetic field.

5. C is correct. Silver and gold are not easily oxidized, so they are weak reducing agents. That means that they will not react with atmospheric gases and corrode.

18. D is correct. The force on the charged particle from the magnetic field is always perpendicular to the particle’s motion, so no work is ever done by the field.

6. B is correct. The resistance of each wire is (0.03 Ω/m)(10 m) = 0.3 Ω. For three resisters in parallel, the formula is 1/R T = 1/R 1 + 1/R 2 + 1/R 3. So 1/RT = 3/0.3, and RT = 0.3/3 Ω = 0.1 Ω.

Passage IV (Questions 19-23) 19. D is correct. AgBr absorbs only wavelengths less than 500 nm. Short wavelength means high frequency.

Passage II (Questions 7-12) 7. A is correct. The oxidation state of nitrogen goes from –3 to 0, so it loses electrons and is oxidized. The oxidation state of chlorine goes from +7 to 0, so it gains electrons and is reduced.

20. D is correct. Te2– is the only doping ion that is sensitive to light of wavelengths between 590 and 630 nm. 21. C is correct. Bromide ion and krypton have the same electron structure, so they are isoelectronic. None of the other statements are true.

8. B is correct. According to the passage, it’s very important to get as much fuel into as small a space as possible, so the volume of a gas makes it a poor choice for rocket fuel. 9. D is correct. The ship must have a speed of

22. C is correct. λ = c/f = 3.0 × 108/5.0 × 1014 Hz = 6.0 × 10–7 m = 600 nm 23. C is correct. If the unreacted AgBr is not removed, it will continue to react when exposed to light. Since the reaction produces elemental silver, which darkens the negative, it will eventually turn black.

2 gR , so

the kinetic energy must be (1/2)m 2 gR 2 = mgR. 10. C is correct. According to the passage, iron is a catalyst, so its job is to increase the rate of reaction.

Free Standing (Questions 24-27) 24. D is correct. If two gases are at the same temperature, then they have the same average kinetic energy.

11. B is correct. The reaction produces heat, so the reaction is exothermic and the enthalpy change is negative. The reaction goes from a solid to many gases, so randomness is increaseing and the entropy change is positive.

25. D is correct. If the process is spontaneous at high temperatures but not at low temperatures, the entropy must be increasing and the enthalpy must also be increasing. So the process must be endothermic.

12. C is correct. Reaction 1 produces twice as many moles of oxygen gas as chlorine gas, so there will be twice as much oxygen available for Reactions 2 and 3. Each of these reactions takes 3 moles of gas to produce 2 moles of solid, so if there is twice as much oxygen gas, there will be twice as much aluminum oxide.

26. C is correct. The solution with the lowest boiling point is the solution that experiences the least boiling point elevation. NaNO3 dissociates into 2 particles while all of the others dissociate into three, so NaNO3 will have the lowest boiling point. 27. B is correct. The 3p subshell fills before the 3d, and

Copyright © 2003 Examkrackers, Inc.

66

there will be 3 electrons in the 3p subshell for phosphorous.

40. B is correct. The air speed is greater at point 1, so the pressure is lower. Air will flow through the burrow from high pressure to low pressure. That’s from the flat hole to the raised hole.

Passage V (Questions 28-34) 28. B is correct. The partial pressure at 8900 m is about one-third the pressure at sea level. That means that at the top of the mountain, a given volume of gas will contain one-third as many molecules. Since the fraction of air made up of oxygen remains constant, there will be onethird as much oxygen in each breath. So 3 normal breaths are required at the top of the mountain to equal a single breath at sea level.

41. D is correct. Volume flow is always the same at all points in a closed system of flow. 42. C is correct. In order for the volume flow rate to remain constant, the smaller the cross sectional area, the greater the speed of the flow has to be. 43. A is correct. Volume flow is equal to Av, or (d2/4)(v). Multiplying the volume flow by the density gets the πρvd 2 mass per second, . 4

29. D is correct. Based on the numbers on the table, or on the equation in the passage, the atmospheric pressure decreases exponentially as the altitude increases.

44. A is correct. The bump on the bottom means greater air velocity underneath, which means that the pressure will be greater on top.

30. A is correct. The molar mass of air is 29 g/mol and 1 mole of gas occupies 22.4 liters at STP, so the density of air at STP is (29 g/mol)/(22.4 mol/L) = 1.3 g/L.

45. D is correct. Since the atmospheric pressure is the same over both entrances to the burrow, changes in the atmospheric pressure will not affect the pressure difference, so the air speed in the burrow will not change.

31. C is correct. The decreasing temperature will tend to decrease volume, so the smaller temperature number (in Kelvin) should be in the numerator. The decreasing pressure will tend to increase pressure, so the larger pressure number should be in the numerator. 32. D is correct. Force = (Pressure)(Area). So F = (10 Pa)(2 m2) = 20,000 N.

Passage IX (Questions 46-50) 46. D is correct. W = change in PE + KE = mgh + 1/2 mv2 = (1000)(10)(9) + (1/2)(1000)(62) = 108,000. You can find h and v from the kinematic equations. h = 1/2 at2 = (1/2)(2)(32) = 9. And v = at = (2)(3) = 6.

5

33. A is correct. From Dalton’s law, the partial pressure is equal to the mole fraction multiplied by the total pressure. Poxygen = (0.21)(524 mmHg) = 110 mmHg.

47. D is correct. The cable experiences the most tension when acceleration is upward or positive. When the acceleration is upward, the tension in the cable is mg + ma. Of the choices given, only Stage 6 has a positive acceleration.

34. B is correct. Water boils when its vapor pressure is equal to the atmospheric pressure. Since the atmospheric pressure is lower at high altitudes, less energy is required to bring the vapor pressure up to atmospheric pressure and the boiling point of water decreases. Water boils at a lower temperature. The lower temperature will require more time to cook the egg and make it hardboiled.

48. A is correct. By looking at Figure 1, we can see that the positive displacement (area under the function and above the x axis) is less than the negative dislacement (area above the function and under the x axis). This means that there is a net negative displacement.

Passage VI (Questions 35-39) 35. A is correct. You can see from Reaction 1 that hydrogen ions are produced in the reaction. If the water is already acidic, then the equilibrium will be driven in the reverse direction, away from HOCl.

49. C is correct. From Figure 1, the velocity is positive and the acceleration (slope) is negative. 50. B is correct. The elevator cable must pull harder than mg in order to accelerate the elevator upward in Stage 1. At constant velocity in Stages 2 and 4, the tension in the cable is equal and opposite to gravity to make the net force zero on the elevator. In Stage 3, the elevator is going up but slowing. This requires a tension less than mg. In Stage 6, the elevator is going down but slowing. This requires tension greater than mg.

36. D is correct. In the solubility product, the coefficients of the products become exponents. 37. B is correct. Use the Henderson Hasselbalch equation. pH = 7.5 + log(0.01)/(0.1) = 7.5 – 1 = 6.5 38. A is correct. (7.1 g/106 g)(103 g/1 L)(1 mol/71 g) = 10–4 mol/L 39. B is correct. Calcium and hydrogen trade partners, so it’s a double replacement reaction. Passage VII (Questions 40-45) Copyright © 2003 Examkrackers, Inc.

67

Free Standing (Questions 51-55) 51. A is correct. Magnesium has a completed s subshell, so all of its electrons are spin paired, making it diamagnetic.

Passage X (Questions 62-66) 62. B is correct. Light ‘refracts’ through a lens. 63. B is correct. In order to invert the image of the slide, the lens must be a converging lens. A converging lens will only invert the image when the object (the slide) is outside the focal distance. p > f

52. D is correct. All of these molecules are non polar, so their only intermolecular forces are London dispersion. London dispersion forces are more prominent between larger molecules. Also, bromine is the only choice that is a liquid at room temperature, so it must have the strongest intermolecular forces.

64. C is correct. The light passes through the lens so the image is real. Only converging lenses produce real images. 65. C is correct. Use the magnification equation. m = –q/p = –300/6 = –50. You can discard the minus sign because the question doesn’t ask about the orientation of the image. The size of the image is (4)(50) = 200 cm = 2 m.

53. A is correct. The Group 1 elements will have larger second ionization energies than the Group 2 elements because their second electrons must be removed from an inner shell. Sodium is higher up the periodic table than potassium, so each of its successive ionization energies will be greater than potassium’s corresponding energy.

66. B is correct. Convection is heat transfer via currents, such as air currents produced by the fan.

54. C is correct. Use Hess’s law. If you reverse the second reaction divide it by two, you will have one mole of CO as your product. Now add the two reactions together, you’ll get the reaction for the standard enthalpy of formation of CO. –393 + 283 = –110

Passage XI (Questions 67-71) 67. C is correct. KE = 1/2 mv2 = (0.5)(80)(625) = 25,000 J = 25 kJ

55. B is correct. Use the oxidation potential for Cr2+ (same as the reduction potential, except positive). Add the two potentials together. 0.41 V + 0.15 V = 0.56 V. Remember, don’t use coefficients when calculating reaction potentials.

68. B is correct. All the potential is eventually converted to kinetic. The potential is proportional to the height, so 16 meters is halfway down and half the potential is lost at that point. 69. C is correct. Horizontal speed remains constant throughout the flight of a projectile. The acceleration due to gravity is also constant. The vertial speed increases until the diver hits the water.

Passage III (Questions 56-61) 56. A is correct. Milk donates a proton to the bicarbonate ion, so it is a Bronsted-Lowry acid.

70. A is correct. Waves in shallow water move more slowly when the water gets more shallow.

57. B is correct. Since carbon dioxide is linear, it is a non polar molecule. Water’s bent shape makes it polar. Water’s polarity makes its intermolecular forces greater, causing it to remain a liquid at higher temperatures than carbon dioxide.

71. B is correct. The diver covers a horizontal distance of d = vt = (3)(2.5) = 7.5 m. The rocks extend 4 m, so the diver clears the rocks by 7.5 – 4 = 3.5 m.

58. B is correct. Carbonate ion has a triangular structure, with one resonant double bond. 59. D is correct. Increased temperature always increases the rate of a reaction. Increased temperature makes gases less soluble.

Free Standing (Questions 72-77) 72. D is correct. Sound travels faster in water than in air even though water is more dense because water is more resistant to compression than air and this factor turns out to be more important for water and air.

60. C is correct. Sodium bicarbonate is a base. All of the other substances are acids, so sodium bicarbonate is the only one that will produce a solution with a pH greater than 7.

73. A is correct. At point A, the track must supply a counter force against gravity and the centripedal force to turn the coaster. At B and C, the counter force against gravity is unnecessary.

61. C is correct. 8.4 grams of sodium bicarbonate is 0.1 mole. According to reactions 1 and 2, each mole of sodium bicarbonate produces 1 mole of carbon dioxide. So 0.1 moles of carbon dioxide is produced. At STP, 0.1 mole of a gas occupies 2.2 liters.

74. A is correct. Use Hooke’s law to find k. F = kx. k = F/x = (40)/(0.01) = 4000 N/m. PE = 1/2 kx 2 = (1/2)(4000)(0.012) = 0.2 J

Copyright © 2003 Examkrackers, Inc.

75. D is correct. Take the torques from the left end of the bar. Clockwis torque is (8)(0.5) + (2)(1) = 6 Nm. Counterclockwise torque is (10)(x). Set the torques equal, 10x = 6. x = 0.6 m.

68

76. D is correct. The formula is L = nλ/2, where n = 1, 2, and 3, and L is the length of the pipe. Notice that you only need to find the first harmonic for the question. 77. D is correct. Greater force would mean greater change in length, so I is correct. Smaller diameter would mean greater stress for the same force, resulting in greater change in length, so II is correct. Greater original length would result in greater change in actual length for the same fractional change in length, so III is correct.

Copyright © 2003 Examkrackers, Inc.

69

Copyright © 2003 Examkrackers, Inc.

70

Answers & Explanations Verbal Reasoning Questions 78 - 137

71

Passage I 78.

79.

80.

The example that “a coal mine was allowed to hire children to pull carts of ore in exchange for a few pence a (12-hour) day, since the starving child was always “free” to refuse, and the subjective value of those shiny coins was presumably much higher to the child in need than it would be to the mine owner’s children” (lines 18-23) implies that the author may believe: A. the British system of contract law is morally superior to the U.S. system. CORRECT: This example is quite drastic an demonstrates a moral absurdity of the U.S. system. It is not rebutted in any way and thus demonstrates that the author has a prblem with the U.S. system. The British system would not result in such an egregious turpitude. B.

the U.S. system of contract law is morally superior to the British system. WRONG: This is the opposite of A, which is correct.

C.

contracts are judged by U.S. courts based upon their fairness.. WRONG: The example does not address this.

D.

contracts are judged by U.S. courts based upon the intentions of the parties. WRONG: The example does not address this.

The author is primarily concerned with demonstrating that: A. the American enforceability of contracts is not as far from the British model as one might think. CORRECT: Of the four answer choices provided, the author is primarily concerned with demonstrating this idea. (Note that there may be other ideas or a thesis in the passage which the author is even more concerned with demonstrating. Yet you are still limited to the four answer choices provided here. This is not unusual in an MCAT.) Despite rhetorical protestations to the contrary, the American courts actually must, and do, decide upon the questions of reasonableness and benefits. B.

there is relatively little rhyme or reason to the enforceability of contracts under American courts. WRONG: The author is not primarily concerned with demonstrating this idea. This is a vague answer. Particularly considering the use of “relatively little” and “rhyme or reason”. Further, the author believes that there is reason to the American court decisions; only that it is probably not what it appears on its face value.

C.

one should exercise caution before entering lightly into a contract. WRONG: The author is not primarily concerned with demonstrating this idea. This is also a rather vague answer. Most importantly, the passage is not provided as a ‘how-to’ guide for entering into contracts. It is meant to clarify the American system by contrasting and comparing it with the British.

D.

the idea of ‘benefit’ is still strongly on the side of the provider under American concepts of enforceability. WRONG: The author is not primarily concerned with demonstrating this idea. Jones v. Star Credit would not support this idea. It is not clear that either party has a particular benefit with regard to contracts.

According to the passage, the real meaning of American court “ideals” such as “freedom of contract” (line 7) is best shown by analysis of the: A. rhetorical and oft-stated differences between American and British law. WRONG: Analysis of this answer would not show the real meaning of “freedom of contract”. It is clear that the author does not believe the rhetoric. B.

early court decisions of contract enforceability. WRONG: Analysis of this answer would not show the real meaning of “freedom of contract”. The early court case, refers to the coal mine and is not indicative of a more current situation or case in Britain and America. This is common knowledge. The Industrial Revolution is long over.

C.

benefits and detriments used by judges to make their determinations. WRONG: Analysis of this answer would not show the real meaning of “freedom of contract”. This answer is to specific. It is encompassed in Answer D, while Answer D also allows for other factors that could be analyzed. Consider a situation where you were allowed to analyze on the “benefits and detriments used by judges to make their determinations” (this answer), while your opposing debater were allowed to analyze the “actual rulings regarding those types of cases” (Answer D). Who would be able to better argue the “real meaning of ‘freedom of contract’”?

Copyright © 2003 Examkrackers, Inc.

72

D.

81.

82.

83.

actual rulings regarding those types of cases. CORRECT: Analysis of this answer would show the real meaning of “freedom of contract”. This is implied rather than explicitly stated. However, the author clearly believes that the “oft-repeated rhetoric” of American courts is not accurate. And that “the concept of benefit influences the law of contract; it is pervasive and persistent ...” (lines 2325). Jones v. Star Credit is a good example, and later, “it becomes necessary for the judge to calculate the benefit at some point” (lines 48-49), which one would presumably see if analyzing the court case.

According to the passage, if there is no calculation of benefit to influence the law of contract, then: A. adults will not be free to negotiate. WRONG: This would not clearly be the result of the supposition in the question. Actually, if there were no calculation of benefit then they would be free to negotiate the most favorable deal possible, disregarding any concepts of benefit being consistent with prices. B.

there will be continual efforts on the part of at least one of the parties to take advantage of the other. WRONG: This would not clearly be the result of the supposition in the question. This idea is not necessarily true, though it is a possible foreseeable outcome.

C.

there is no way to ensure a contract’s intrinsic fairness. CORRECT: This would clearly be the result of the supposition in the question. The “concept of benefit influences the law of contract; it is pervasive and persistent, as it must be to ensure contracts’ intrinsic fairness” (lines 23-26).

D.

the ‘law’ will be nothing more than the whims of the parties involved. WRONG: This would not clearly be the result of the supposition in the question. This answer is vague and subject to subjective interpretations.

The passage suggests that an automobile loan company would not charge extremely exorbitant rates to customers because these rates might be: A. seen by the company to be immoral. WRONG: This is not suggested as a possible answer to the question. B.

illegal under popular court decisions. WRONG: This is not suggested as a possible answer to the question. There is no information to suggest that rates would necessarily be illegal.

C.

considered unconscionable by the courts. CORRECT: This is suggested by the example of the Joneses the passage.

D.

expressly forbidden by U.S. statute. WRONG: This is not suggested as a possible answer to the question. There is no information to suggest that rates would necessarily be illegal.

According to the early court “dicta” (lines -22), a contract should always be enforced as written because: A. the contract might have been worth a great deal to a party willing to accept the risk. CORRECT: “In early court decisions’ dicta, this principle was justified with the pretext that, since a contract’s subjective worth to any party might be far more than the objective monetary value of the benefit it provides, no judge should impose his own valuation on the benefit, and find it wanting” (lines 12-16). B.

the alternative is that no contract is worth more than the paper it is printed upon. WRONG: This answer has nothing to do with the question and is not supported in the passage.

C.

the concept of benefit should rule the law of contract. WRONG: Not according to early court “dicta”. This is a British concept which the author feels that American courts follow despite their “oft-repeated rhetoric” to the contrary.

D.

a judge is in a position to make a good determination of the contract’s original value. WRONG: This answer is diametrically opposed to the correct idea. “In early court decisions’ dicta, this principle was justified with the pretext that, since a contract’s subjective worth to any party might be far more than the objective monetary value of the benefit it provides, no judge should impose his own valuation on the benefit, and find it wanting” (lines 12-16).

Copyright © 2003 Examkrackers, Inc.

73

84.

Which of the following opinions would the author be most likely to endorse? A. A written contract should bind the parties regardless of perceptions of unconscionability. WRONG: The author would not be most likely to endorse this opinion. The author does have an opinion in the passage. “But in many ways – some obvious, some veiled – the concept of benefit influences the law of contract; it is pervasive and persistent, as it must be to ensure contracts’ intrinsic fairness” (lines 23-26). B.

The American standards for contracts are actually the same as the British standards. WRONG: The author would not be most likely to endorse this opinion. This answer is too simplistic. Consider asking, “Are they ‘actually the same’ in all ways?”

C.

Contracts should be held to a standard of ‘reasonableness’. CORRECT: The author would be most likely to endorse this opinion. The author does have an opinion in the passage. “But in many ways – some obvious, some veiled – the concept of benefit influences the law of contract; it is pervasive and persistent, as it must be to ensure contracts’ intrinsic fairness” (lines 23-26).

D.

Inability to understand if a contract is fair or not should not be an excuse for breaking it. WRONG: The author would not be most likely to endorse this opinion. Though the author does see some irony in the Jones decision, he does also seem to believe that there have to be standards of fairness which presumably would include both parties basically understanding the contract. This is not the best answer.

Passage II 85.

86.

The author’s characterization of tanning, suggests that the retort/comment “adequate recreational time to spend outdoors, out of the office” (lines 10-11) means that nowadays a tan symbolizes: A. health. WRONG: This is not the reason. B.

status. CORRECT: This is the suggested meaning of what tanning symbolizes ‘nowadays’. The author draws a metaphor between what tanning meant during the time of the Roman Empire and what it means now. At that time “their pallor was more a status symbol than a beauty enhancement” (lines 4-5). Further, this answer can encompass the ideas of ‘beauty’ and ‘wealth’. It is an inclusive answer. ‘Nowadays’ “a tan suggests the person has adequate recreational time to spend outdoors, out of the office” (lines 10-11).

C.

beauty. WRONG: This is not the suggested meaning of what tanning symbolizes ‘nowadays’. The author draws a metaphor between what tanning meant during the time of the Roman Empire and what it means now. At that time “their pallor was more a status symbol than a beauty enhancement” (lines 4-5).

D.

a farming background. WRONG: This is not the suggested meaning of what tanning symbolizes ‘nowadays’. This is a poor answer choice. Any careful reading of the passage would tell you that this was not the author’s meaning.

On the basis of the passage, it is reasonable to conclude that: A. most people use sunscreens when outdoors. WRONG: This is not a reasonable conclusion on the basis of the passage. “Most” people? They might or they might not. B.

sun tanning is popular today. CORRECT: This is a reasonable conclusion on the basis of the passage. Beginning with the first sentence, “Sun tanning was not always popular”, which leaves us with the impression that the author believes that tanning is ‘now’ popular, the author emphasizes this idea. “For better or worse, our culture now associates tanning with good health, and pallor with illness” (lines 15-16). It seems that people will continue tanning (while it is popular and a status symbol) even though it is not good for you.

C.

farmers are likely to get skin cancer. WRONG: This is not a reasonable conclusion on the basis of the passage. This assumes that the farmers are not wearing hats or sunscreen, and a myriad of other ideas.

Copyright © 2003 Examkrackers, Inc.

74

D.

87.

88.

89.

90.

tanning promotes skin blemishes. WRONG: This is not a reasonable conclusion on the basis of the passage. A careful reading of the passage will find no support for this answer.

An important comparison is made in the passage between: A. skin pallor and health. WRONG: This is not an important comparison. The idea is mentioned briefly in the first paragraph in regards to the changing perceptions of society. Yet this comparison could be removed with little change in the passage’s main ideas. Pardon the pun, but this comparison ‘pales’ in comparison to Answer D. B.

Rome and modern society. WRONG: This is not an important comparison. Again, this comparison is rather unimportant in relation to the emphasis of the paragraph.

C.

outdoor and indoor tanning. WRONG: This is an important comparison, but it is not as important as Answer D.

D.

UV A and UV B rays. CORRECT: This is an important comparison. This is the most important comparison provided in the answer choices. The information is referred to throughout the majority of the passage.

Which of the following findings would most compromise the author’s conclusions about sun tanning? A. For a person with unblemished skin, tanning offers almost no cosmetic benefits. WRONG: This finding would not most compromise the author’s conclusions about tanning. It does not really compromise the findings at all. B.

From the time of the Roman Empire, the number of people diagnosed with skin cancer has hardly risen. WRONG: This finding would not most compromise the author’s conclusions about tanning. This would be a very convulated compromise, requiring many assumptions.

C.

Though it takes longer, one can acquire the same dark rich tan when using an SPF 15 sunscreen. CORRECT: This finding would most compromise the author’s conclusions about tanning. “Contrary to popular belief, tanning with SPF 15 sunscreen is still possible, but it requires longer exposure, and produces paler skin” (lines 42-44). Thus the author points out that this Answer/finding is actually not possible.

D.

In a very few areas of the world, the sun’s rays become strongest sometime after 2:00 p.m. WRONG: This finding would not most compromise the author’s conclusions about tanning. The idea that “in a very few areas of the world” is very specific and does not really compromise the author’s generalities regarding sun intensity.

The author’s discussion of the relationship of skin shades to activity and wealth (lines 1-11) implies that: A. Roman women would choose not to work if given the choice. CORRECT: This is implied by the author’s discussion of the rlationship of skin shades to activity and wealth. “Since most of the population earned a meager living by farming outdoors, a woman’s pale skin advertised publicly that she was wealthy enough not to work” (lines 5-7). B.

the number of Roman farmers was proportionally small. WRONG: This is not implied by the author’s discussion of the rlationship of skin shades to activity and wealth.

C.

all Roman women helped as farmers. WRONG: This is not implied by the author’s discussion of the rlationship of skin shades to activity and wealth. It is clear from the quote (lines 5-7), that “most” (i.e. not “all”) the population worked as farmers.

D.

pale skin was a mark of beauty for Roman women. WRONG: This is not implied by the author’s discussion of the rlationship of skin shades to activity and wealth. “During this period, their pallor was more a status symbol than a beauty enhancement” (lines 3-5). This means that it was more a badge of some sort, or a uniform. It is not clear that tanning was a mark of beauty for Roman women. This is a big assumption.

The passage suggests that most people who are using sunscreens:

Copyright © 2003 Examkrackers, Inc.

75

91.

A.

should not use an SPF higher than 15. WRONG: This is not suggested. In fact, “Doctors recommend at least SPF 15 to protect against damage” (lines 38-39).

B.

will probably peel rather than tan. WRONG: This is not suggested.

C.

will be enjoying the protection of both PABA and benzophenones. CORRECT: This is suggested. “Most sunscreens contain both types of blockers” (lines 36-37).

D.

may not be aware of the risks they are taking. WRONG: This is not suggested.

According to the passage, the potential damaging effects of the sun’s rays on unprotected skin would be determined by taking into account: A. whether the exposure were gradual or intense. WRONG: The potential damaging effects of the sun’s rays would not be determined by taking this into account. Many other factors would have to be considered also. This answer is too specific, and is, in fact, covered in Answer D. B.

the type of sunscreen the person had applied, and when they had applied it. WRONG: The potential damaging effects of the sun’s rays would not be determined by taking this into account. The question specifies “unprotected skin”.

C.

the amount and intensity of the UV B rays on the person. WRONG: The potential damaging effects of the sun’s rays would not be determined by taking this into account. In fact, UV A rays also damage the skin according to the passage.

D.

the amount and intensity of the UV B and UV A rays on the person. CORRECT: The potential damaging effects of the sun’s rays would be determined by taking this into account. This would obviate the need to determine when the exposure occurred, how long the exposure occurred, etc.

Passage III 92.

93.

Assume that two groups of professional bodybuilders undergo similar heavy workouts every day for two months with no rest. Although the building of additional muscle mass is successful among Group A, no increase occurs in Group B. Which of the following hypotheses about this outcome is the most plausible on the basis of the information cited? A. Group A was probably composed of mostly mesomorphs. CORRECT: This hypothesis is the most plausible on the basis of the information cited and the assumption. Notice that there are two ‘softeners’ in this answer rendering it more flexible; “probably” and “mostly”. B.

It is likely that the diet of the Group B did not include enough carbohydrates. WRONG: This hypothesis is not the most plausible on the basis of the information cited and the assumption.

C.

Group A was probably more experienced at bodybuilding. WRONG: This hypothesis is not the most plausible on the basis of the information cited and the assumption. The assumption tells us that both groups “undergo similar heavy workouts”. There is no passage information that would tell us that given similar workouts, more experienced bodybuilders might make greater muscular gains.

D.

Group B was probably exercising too intensely to make muscular gains. WRONG: This hypothesis is not the most plausible on the basis of the information cited and the assumption. The assumption tells us that both groups “undergo similar heavy workouts”. “Intensity” is never fully defined in the passage, and we are not free to assume that “exercising too intensely” would diminish muscular gains.

Researchers state that evidence of hyperplasia, or actual splitting of the individual muscle cells to create new cells, can be found in biopsies from the shoulders of Olympic swimmers who display larger shoulder muscles. This statement tends to support the author’s indication that: A. mesomorphs are not the only group that can attain larger muscles. WRONG: The statement does not tend to support this indication of the author’s. Perhaps the swimmers mentioned in the statement are mesomorphs.

Copyright © 2003 Examkrackers, Inc.

76

94.

95.

96.

B.

it is possible to increase the number of muscle fibers through repeated stress. CORRECT: The statement does tend to support this indication of the author’s. “Some researchers also speculate that the number of muscle fibers can be increased through repeated stress.” (lines 14-16).

C.

when a muscle is placed under stress it may tear and unravel. WRONG: The statement does not tend to support this indication of the author’s. It is not at all clear that this would be related to “hyperplasia”.

D.

after exercise, muscle cells may grow back more thickly than before. WRONG: The statement does not tend to support this indication of the author’s. It is not at all clear that this would be related to “hyperplasia”.

Suppose it is discovered that diet is as important to ectomorphs as it is to mesomorphs. Which of the following statements would be most supported by this discovery? A. Diet is only important to mesomorphs. WRONG: The discovery suggests that diet is important to all. B.

Ectomorphs will find it difficult to gain weight by eating heavily. WRONG: This could be supported or contradicted by the discovery. Perhaps the diet is wrong, so diet is important. Or, perhaps the diet is correct, so diet didn’t work.

C.

It would support the claim that simply eating heavily will not help ectomorphs gain weight. WRONG: This is how the new information would affect the author’s claims. “Such “hardgainers,” as they are called, find it difficult even to gain weight by gorging, much less put on muscle by the usual methods” (lines 46-49).

D.

A special diet is important to bodybuilding ectomorphs. CORRECT: The discovery says that the diet is important to everyone.

Given the information in the passage, Japanese sumo wrestlers, who are mostly interested in increasing their size and strength, most likely do not participate in a great deal of cardiovascular exercise because: A. this type of exercise might cause them to lose weight. CORRECT: This is most likely, given the information in the passage. We know from the passage that cardiovascular exercise may cause an individual to lose weight, and this answer is directly responsive to at least the aspect of ‘increasing size’ offered by the question. B.

they are predominantly mesomorphs. WRONG: This is not most likely, given the information in the passage. There is not enough additional information in the question about sumo wrestlers to come to this conclusion. Additionally, it brings a new element into the equation; bodytypes. While Answer A is directly responsive to the new information in the question.

C.

they may even find it difficult to gain weight through gorging. WRONG: This is not most likely, given the information in the passage. There is not enough additional information in the question about sumo wrestlers to come to this conclusion. Additionally, it brings a new element into the equation; diet. While Answer A is directly responsive to the new information in the question.

D.

they are more interested in wrestling than in bodybuilding. WRONG: This is not most likely, given the information in the passage. There is not enough additional information in the question about sumo wrestlers to come to this conclusion.

Implicit in the statement that, “To the extent that conventional bodybuilding wisdom is unified, it espouses several common points” (lines 17-18), is the idea that: A. most of what is involved in bodybuilding can be distilled into several key points. WRONG: This idea is not implicit in the statement. This idea differs markedly from the statement in that it implies that bodybuilding is rather simple to explain. While the statement is telling us that bodybuilding only shares several commonalities. B.

bodybuilding ideas and concepts are essentially agreed upon in the weightlifting community. WRONG: This idea is certainly not implicit in the statement. The statement tells us that it is arguable that there is much agreed upon at all in bodybuilding.

Copyright © 2003 Examkrackers, Inc.

77

97.

C.

beyond what the author has expanded upon in the passage, there is little agreement. WRONG: This idea is not implicit in the statement. There may be more points that are agreed upon. It is not clear that the author has chosen all of the agreed upon points.

D.

the ideas and practices within the bodybuilding community are varied. CORRECT: This idea is implicit in the statement. The statement tells us that it is “arguable” that bodybuilding wisdom is “unified”.

According to the passage, one/some of the most successfully used type of exercises for professional bodybuilders is/are: A. negative movements. WRONG: This is not one of the most commonly used type of exercises. Negative movements are only indicated as being used by “intermediate-to-advanced” bodybuilders. B.

isolation movements. WRONG: This is not one of the most commonly used type of exercises. Ectomorphs, for one, are advised not to used this type of exercise.

C.

triple-angle training. WRONG: This is not one of the most commonly used type of exercises. This type of training is only suggested for ectomorphs.

D.

compound movements. CORRECT: This is one of the most commonly used type of exercises. These movements are advised for ectomorphs (line 64) and professional bodybuilders (line 24).

Passage IV 98.

Which of the following suppositions is most clearly believed by the author? A. From an early age, one can see that Edward Gibbon was destined for greatness. WRONG: This supposition is not most clearly believed by the author. The author believes that Gibbon’s book is one of the “greatest historical pieces ever written”, but there is no implication of ‘destiny’. B.

Gibbon’s Decline and Fall is still the preeminent historical series on the fall of the Roman Empire. CORRECT: This supposition is most clearly believed by the author. “the Decline and Fall remains one of the greatest historical pieces ever written about the fall of the Roman Empire” (lines 3-5).

C.

Without the harsh guidance of his father, Gibbon might not have been able to produce his opus. WRONG: This supposition is not most clearly believed by the author. There is no support for this supposition.

D.

Gibbon devoted his entire life to producing the Decline and Fall. WRONG: This supposition is not most clearly believed by the author. There is no support for this supposition.

Copyright © 2003 Examkrackers, Inc.

78

99.

100.

101.

102.

The contention that “He must discover the inevitable mixture of error and corruption which she contracted in a long residence upon earth, among a weak and degenerate race of beings” (lines 58-61) can most justifiably be interpreted as support for the idea that: A. A theologian had a more difficult job than a historian. WRONG: The contention cannot most justifiably be interpreted as support for this idea. Actually, the opposite is true. “A more melancholy duty is imposed on the historian” (lines 56-58). B.

Rome was a society of weak and degenerate people. WRONG: The contention cannot most justifiably be interpreted as support for this idea. Line 34 tells us that Gibbon believed the Romans a “great people” of “genius”.

C.

Religion could be twisted by men over time. CORRECT: The contention can most justifiably be interpreted as support for this idea. The “she” of the indented quotation is “Religion” with a capital “R” (line 55). This “Religion,” personified as a female would contract “the inevitable mixutre of error and corruption” from her long residence on earth among weak and degenerate human beings.

D.

Gibbon believed that God was probably a woman. WRONG: The contention cannot most justifiably be interpreted as support for this idea. This is not at all clear. Further, the female reference is to “Religion” with a capital “R” (line 55), not God.

The assertion that Gibbon believed that Rome would have been better off if Christianity had never been allowed within the boundaries of the Empire is NOT clearly consistent with the information about: A. Gibbon’s father’s religious beliefs and background. WRONG: This is not information from the passage. B.

Christianity continually dividing itself into unnumerable sects. WRONG: The assertion and this information seem to have little relation to one another.

C.

Rome falling because religious toleration was disregarded. CORRECT: The assertion is not clearly consistent with this information. “Gibbon made the claim that Rome also fell because these virtues [among them religious toleration] were disregarded” (lines 73-75). The assertion, without further information, seems to conflict with this information.

D.

the “scandal of the pious Christian”. WRONG: The assertion and this information seem to have little relation to one another.

Which of the following sequences most clearly exemplifies Gibbon’s changing religious beliefs throughout his life? A. Catholic, Protestant, Aufklarung WRONG: Gibbon could not have been Catholic first, since he converted to Catholicism at the age of sixteen (lines 12-13). B.

Protestant, Calvinist, Catholic WRONG: Calvinism was “connected with” Aufklarung in some way (lines 23-25).

C.

Aufklarung, Catholic, Protestant WRONG: This is backwards.

D.

Protestant, Catholic, Aufklarung CORRECT: This most clearly exemplifies the sequence of Gibbon’s religious beliefs according to the passage.

Which passage information provides the strongest reason to suppose that Gibbon’s work was considered strong scholastically, but assailed theologically (lines 62-69)? A. Gibbon’s extensive use of secondary sources and the publication of Vindication. CORRECT: This does information provides hte strongest reason. The first paragraph is information which the author provides to support his great admiration for Gibbon and his work. This includes mentioning Gibbon’s voluminous references. Lines 66-69 refer to the publication of Vindication “which was a pamphlet in response to the people who commented upon his critique of Christianity.”

Copyright © 2003 Examkrackers, Inc.

79

103.

104.

B.

The internation recognition received by the work and his expulsion from Oxford. WRONG: There is no strong reference to “international recognition” and the expulsion from Oxford seems to have been prior to Gibbon’s writings.

C.

The attack by Davis and the later recognition Gibbon received from Oxford. WRONG: Gibbon was not recognized by Oxford, but forced to leave Oxford. This seems to have been prior to the attack by David.

D.

Gibbon’s retreat to Aufklarung and his political successes. WRONG: Aufklarung seems to have had little to do with Gibbon’s work.

On the basis of the passage, it is reasonable to conclude that: A. Gibbon believed Rome would have continued to prosper in the absence of Christianity. WRONG: This is not a reasonable conclusion. B.

Gibbon believed the Romans should have been more tolerant of the Christians. WRONG: This is not a reasonable conclusion. “Gibbon believed that Rome would have been better off if Christianity had never been allowed within the boundaries of the Empire” (lines 29-32).

C.

Gibbon strove to make England more Roman-like. WRONG: This is not a reasonable conclusion. Gibbon promoted virtues which he believed had led to the fall of the Roman Empire “because these virtues were disregarded.”

D.

Gibbon would have applauded Rome’s general tolerance of many different gods and many different religions besides Christianity. CORRECT: This is a reasonable conclusion. Based upon the last paragraph, it seems that Gibbon was for “religious toleration” (though not for the Christians who should have never been allowed within the Roman Empire). There is nothing within the passage to lead us to believe that the new information (offered in the question) regarding “Rome’s general tolerance” is inaccurate.

Which of the following assertions does the author support with an example? I. There were theological attacks on Gibbon’s work. CORRECT: Lines 62-69, and Davis from Oxford are offered as examples. II. At least once, Gibbon responded in writing to theological criticisms. CORRECT: Again, in the same paragraph as Answer I, Gibbon responded to a theological criticism by publishing Vindication. III. Gibbon’s father was an active member in the protetant movement. WRONG: There are no examples in support of this assertion. A. B.

I only I and II only CORRECT: See above answer explanations.

C. D.

II and III only I, II, and III

Passage V 105.

Given the information in the passage, if the word “brethren” (line 57) had been used by Helvidius, in which of the following examples would it have been most likely to have appeared? A. Jesus often referred to his numerous disciples as his “brethren”. WRONG: If the word had been used by Helvidius it would not have been most likely to have appeared in this example. This does not further Helvidius’ argument. This may have been a part of Helvidius use of the word, but it is more likely, given the context of Helvidius and Jerome’s arguments in the passage, that Helvidius would have been using the word to strengthen his arguments that Mary was not a virgin. B.

Speaking to men of all races, creeds, and nationalities as “brethren” showed Jesus’ love of all men. WRONG: If the word had been used by Helvidius it would not have been most likely to have appeared in this exam-

Copyright © 2003 Examkrackers, Inc.

80

ple. Helvidius would have been using the word to strengthen his arguments that Mary was not a virgin. This example does nothing to further his argument.

106.

107.

108.

C.

The word “brethren” connotes a closeness and familiarity with all members of an extended family. WRONG: If the word had been used by Helvidius it would not have been most likely to have appeared in this example. This example actually weakens Helvidius’ argument and would more likely have been used by Jerome.

D.

It is apparent from the word “brethren” that Jesus had brothers. CORRECT: If the word had been used by Helvidius it would have been most likely to have appeared in this example. This example furthers the arguments of Helvidius and is the most likely of the examples to have been used by him.

If the characterization of Charles Mierow’s statement is correct (lines 6-8), and it is understood that “cannonizing” is the process whereby a person is recognized as a saint, then it can be inferred that: A. By studying extremely hard, one can be declared a saint. WRONG: This cannot be inferred from Mierow’s statement and an understanding of “cannonizing”. B.

Besides Jerome, no one has ever been declared a saint based on their scholarship. CORRECT: This can be inferred from Mierow’s statement and an understanding of “cannonizing”. “Charles Mierow states that Jerome is the unique example of a man who was canonized for his scholarship” (lines 6-8). The keyword here is “unique”.

C.

There is only one saint who was a scholar. WRONG: This cannot be inferred from Mierow’s statement and an understanding of “cannonizing”. This answer is not at all the same as Answer B. There may have been many other saints who were scholars, but cannonized for reasons other than their scholarship.

D.

Scholars, such as Jerome, usually have great difficulty becoming saints. WRONG: This cannot be inferred from Mierow’s statement and an understanding of “cannonizing”. They may or they may not. This is not the best answer.

Which of the following statements, if true, would most directly challenge the principles of Mariology? A. Helvidius’ scriptural quotations were found to be contextural in their references. CORRECT: This statement would most challenge the principles of Mariology. It seems that the main difference between the extensive documentary arguments of Jerome and Helvidius was Jerome’s argument that (lines 51-53) Helvidius had taken words and phrases out of their proper context. B.

Jerome actually agreed with most of what Carterius had written. WRONG: This statement would not most challenge the principles of Mariology. Jerome probably did agree with most of what Carterius had written.

C.

Jerome’s work had far fewer references and scriptural quotes than Helvidius’. WRONG: This statement would not most challenge the principles of Mariology. This was not the crux of their arguments. This may have diminished Jerome’s work somewhat in comparison to Helvidius, but it was Jerome’s emphasis on Helvidius’ taking quotes and phrases out of context that the author feels was most significant.

D.

Mary remained a virgin throughout her life. WRONG: This statement would not most challenge the principles of Mariology. This is a basic tenet of Mariology.

Given that Mary was believed by Christians to be married to a man named Joseph, what would be the author’s most likely response to some one who questioned “the superiority of perpetual virginity over marriage” (lines 6-8)? Remember with these types of questions that the author’s would not be most likely to admit mistakes, or acquiesce in an argument, but instead would choose an answer which would coincide with or rehabilitate his arguments and perspective. A. St. Jerome did not perceive Mary to be ‘married’ to Joseph. WRONG: This would not be the author’s most likely response to the question. It is clear from the passage that Jerome was a Christian and therefore would have shared their basic beliefs. Thus he would not have argued that she wasn’t married. B.

By ‘marriage’, St. Jerome meant sexual intercourse between a man and a woman. CORRECT: This would be the author’s most likely response to the question. Given that Jerome felt that Mary was

Copyright © 2003 Examkrackers, Inc.

81

a perpetual virgin, yet probably also believed, as a Christian, that she was married, he would most likely have ‘clarified’ what as meant by ‘marriage’. This idea of clarifying the context of a word or phrase as likely to have been Jerome’s most likely response is supported in the passage by the fact that this is exactly how Jerome argued against Helvidius.

109.

110.

C.

They were actually one in the same in this instance. WRONG: This would not be the author’s most likely response to the question. If they are one in the same then one cannot be superior to the other.

D.

These ideas would not apply to Mary and her husband. WRONG: This would not be the author’s most likely response to the question. This would have been a weak response and not likely to have been offered by Jerome.

Which of the following statements is the most reasonable conclusion that can be drawn from the author’s description of Jerome’s treatise on Mariology (lines 38-53)? A. Carterius’ writings were the only ones previous to Jerome’s on this subject. WRONG: This is not the most reasonable conclusion that can be drawn from the author’s description. There is no support for this answer. Beware of words like “only” unless they are explicitly supported within the passage. B.

Prior to Jerome, only Helvidius and Carterius had written on this subject. WRONG: This is not the most reasonable conclusion that can be drawn from the author’s description. There is no support for this answer. Beware of words like “only” unless they are explicitly supported within the passage.

C.

Helvidius did not use the Vulgate for his scriptural references. WRONG: This is not the most reasonable conclusion that can be drawn from the author’s description. Maybe he did, and maybe he didn’t. Simply because he interpreted the meaning of words and phrases differently than Jerome does not mean that they didn’t both use the same references.

D.

There had been documents written previously on this subject, though not in Latin. CORRECT: This is the most reasonable conclusion that can be drawn from the author’s description. “[Jerome’s] is the first treatise in Latin especially devoted to Mariology” (lines 44-45).

The word partum (line 61) is used in the sense of: From the passage, “The result of the work is . . . the doctrine of Mary’s virginity in pre partum and post partum that it was never again seriously doubted in Roman circles” (lines 58-62). A. sexual intercourse. WRONG: The word is not used in this sense. The statement and line refer to Jerome’s triumph in perpetuating Mary’s virginity. Thus, “post sexual intercourse” would mean that she was not a virgin. B.

birth. CORRECT: The word is used in this sense.

C.

time-of-the-Bible. WRONG: The word is not used in this sense.

D.

time-of-Jerome’s-treatise. WRONG: The word is not used in this sense.

Passage VI 111.

Evidence shows that identical twins who have been separated since birth display remarkable similarities in their behaviors, likes, and dislikes. This fact tends to support the hypothesis concerning the Interaction Model because: Notice that the “evidence” in the question would actually seem to best support the Trait Model. But this is not a possibility. The question is specifically structured to force you to think in a non-standard fashion and use the passage information in a new way. A. only this model accounts for the inherently similar traits of the twins. WRONG: This is not why the fact tends to support the hypothesis. The “only” in this answer immediately renders it incorrect since the Trait Model is readily provided support by the fact.

Copyright © 2003 Examkrackers, Inc.

82

112.

113.

114.

B.

it is likely that the twins were raised in very similar ways. WRONG: This is not why the fact tends to support the hypothesis. This answer is probably not likely, and it is certainly speculative.

C.

it is the social interactions of the twins that are noticeably similar. WRONG: This is not why the fact tends to support the hypothesis. It does not use any of the passage information. If you don’t understand the answer choice, then don’t choose it.

D.

this model emphasizes both traits and environment. CORRECT: This is why the fact tends to support the hypothesis. From the passage, “the Interactional Model, aptly addresses the joint role of traits and environment” (lines 60-61). Certainly, as mentioned above, the Trait Model is better supported. However, if you are forced to answer why the fact tends to support the Interaction Model, then this is clearly the best answer.

Which of the following scientific advances would most seriously challenge the hypothesis involving the “Goodness of Fit model? A. Association between genetic predispositions of both the child and the caregiver. WRONG: This advance would not most seriously challenge the hypothesis of the model. It is not clear what thsi answer means. Secondly, including the caregiver in this answer lends at least some support to the “interaction” aspect required of the Goodness of Fit model. B.

Proof of ADHD occurring in many children who had suffered childhood head injuries. WRONG: This advance would not most seriously challenge the hypothesis of the model. How many is “many”? It is certainly not “all”, and probably is not most “most”. This is not as challenging to the model as Answer D.

C.

Confirmation of studies indicating that environmental stresses contribute markedly to ADHD. WRONG: This advance would not most seriously challenge the hypothesis of the model. This actually supports the model.

D.

Further correlation of genetic predispositions as the major factor in ADHD. CORRECT: This advance would most seriously challenge the hypothesis of the model. “[The Goodness of Fit] model emphasizes the interaction between the child’s temperament and the type of care the child receives” (lines 6465). Thus it is challenged by anything which supports genetic predispositions.

According to the passage information, what would happen in an environment where children were exhibiting severely antisocial behavior? A. The children would most likely be diagnosed as suffering from ADHD. WRONG: It is not likely that this would happen as a result of exhibiting severely antisocial behavior. The question is purposely vague. This answer uses passage information, yet it is too specific. What if the children were gang members and juvenile delinquents? B.

The caregivers of these children might not be able to put up with their behavior. CORRECT: It is likely that this would happen as a result of exhibiting severely antisocial behavior. The question is purposely vague. Yet in any given situation, this answer would be appropriate. “it is possible to see how even the most tolerant parent can be driven to intolerance if the child displays an unbearable amount of fussiness, impulsivity, and inattention” (lines 76-79).

C.

The caregivers would likely modify their own behavior to match that of the child. WRONG: It is not likely that this would happen as a result of exhibiting severely antisocial behavior. The question is purposely vague. This answer uses passage information, yet it is too specific, and can be rather silly. What if the children were gang members and juvenile delinquents?

D.

A study would be undertaken to determine if the child were suffering from Fetal Alcohol Syndrome. WRONG: It is not likely that this would happen as a result of exhibiting severely antisocial behavior.

Which of the following suppositions is most clearly believed by the author? A. The Trait Model is the poorest paradigm for use in studying ADHD. CORRECT: This supposition is most clearly believed by the author. The passage clearly takes us through a progression of models which is not chronological, but is based upon a heirarchy of the author’s belief in their credibilities or lack thereof.

Copyright © 2003 Examkrackers, Inc.

83

115.

116.

117.

B.

The Environment Model is the best paradigm for use in studying ADHD. WRONG: This supposition is not most clearly believed by the author. In fact, it is probably true that the author feels that the last model is the most appropriate (lines 60-62).

C.

It is relatively easy to prove or disprove the Trait Model as an ADHD paradigm. WRONG: This supposition is not most clearly believed by the author. Lines 18-24 belie this answer. If this answer were true, then the author would have clearly said that the Trait Model is either proven or disproven.

D.

It is very difficult to discern the difference between children suffering from Fetal Alcohol Syndrome and ADHD. WRONG: This supposition is not most clearly believed by the author. Fetal Alcohol Syndrome is merely offered as another example wherein it is difficult to discern whether what is going on is purely genetic or not.

For which of the following of the author’s assertions is NO support provided in the passage? A. Changes in the course of ADHD are allowed for in the Environmental Model. WRONG: There is support provided for this assertion. “Unlike the Trait Model, the Environmental Model allows for changes in the course of ADHD” (lines 50-51). B.

It is easier to change the child than the child’s environment. CORRECT: This is an assertion of the author’s and there is no support provided for it within the passage. “But given the fact that it is much more difficult to change a child’s environment than to change the child ...” (lines 56-57).

C.

Without further study, the most accurate paradigm cannot be determined. WRONG: This is not an assertion of the author’s.

D.

Children with Fetal Alcohol Syndrome will usually also have ADHD. WRONG: This is not an assertion of the author’s.

Regarding the relative accuracy of the ADHD “paradigms”, the passage strongly implies that: A. the Environmental Model suffers from a purity of form. WRONG: This is not strongly implied regarding the relative accuracy of paradigms. B.

the Trait Model is no longer widely used. WRONG: This is not strongly implied regarding the relative accuracy of paradigms.

C.

each has its place, either singularly, or in tandem. WRONG: This is implied at all regarding the relative accuracy of paradigms.

D.

the Interactional Model is the most appropriate. CORRECT: This is strongly implied regarding the relative accuracy of paradigms. The “most appropriate” is a subjective value judgement required of the author. It is clear that the author believes the Interactional Model to be the most “apt” (lines 60-64).

According to the passage, a central problem to be solved with regard to ADHD is that of: A. adapting the child to his environment. WRONG: This is not a central problem to be solved with regard to ADHD. B.

determining who is at fault. WRONG: This is not a central problem to be solved with regard to ADHD.

C.

making an accurate diagnosis. CORRECT: This is a central problem to be solved with regard to ADHD. This is mentioned at lines 18-21 and alluded to throughout the passage.

D.

modifying the treatment strategies. WRONG: This is not a central problem to be solved with regard to ADHD.

Copyright © 2003 Examkrackers, Inc.

84

Passage VII 118.

119.

120.

121.

What distinction is implied in the passage between white soldiers and black soldiers, respectively? A. Less and better prepared for war CORRECT: This distinction is implied in the passage. “black soldiers were ... many times, better prepared for war than were white soldiers” (lines 8-10). B.

More and less strongly built WRONG: This distinction is implied in the passage, but it is not respective. Higginson considered the black soldiers more muscular.

C.

Less and more resistant to diseases WRONG: This distinction is implied in the passage, but it is not respective. Higginson considered the black soldiers less resistant to diseases.

D.

Poorer and better officers among them WRONG: This distinction is not implied in the passage.

The author’s argument that black soldiers were also fighting for the opportunity to be ‘compared’, depends on the acceptance of which of the following premises? A. Higginson was able to recognize these subtleties and accurately document them. WRONG: The author’s argument does not depend on the acceptance of this premise. It is the author who is claiming that Higginson (who apparently was actually there) had failed to notice something. B.

Black soldiers were, in many ways, superior to the white soldiers. WRONG: The author’s argument does not depend on the acceptance of this premise. It has nothing to do with it.

C.

The author is better able to recognize these nuances than Higginson was. CORRECT: The author’s argument does depend on the acceptance of this premise. “Higginson points out that the black soldiers were fighting for their freedom, failing to see that at least the black soldiers themselves recognized that they were also fighting for the opportunity to be ‘compared.’” (lines 34-37). The author, who presumably was not there, but at the least has not provided anything that would further credit his own perspective, is arguing that Higginson may not have noticed something!

D.

The Civil War was fought in an effort to free the slaves. WRONG: The author’s argument does not depend on the acceptance of this premise. This premise has nothing to do with the argument.

The author’s discussion of the relationship of resistance to illness to the soldier’s living conditions (lines 46-61) implies that: A. the average citizens of the period thought little of the black soldiers. WRONG: The author’s discussion of the relationship does not imply this answer. B.

the religious beliefs of the black soldiers helped them recover quickly. WRONG: The author’s discussion of the relationship does not imply this answer.

C.

without the aid of the white soldiers, sick black soldiers would have died quickly. WRONG: The author’s discussion of the relationship does not imply this answer.

D.

Higginson was a white officer. CORRECT: The author’s discussion of the relationship implies this answer. “In an observation which would have been shared by his white fellows ... Higginson states that blacks were less resistant to diseases than white soldiers” (lines 47-51).

What does the argument that the average citizen probably believed that there were tremendous differences between the white and black soldiers, but would have been unable to compare the two, imply about the reasons for the North wanting to abolish slavery? A. The differences between the two would not have been known to the average citizen. WRONG: The argument does not imply this about the reasons for the North wanting to abolish slavery.

Copyright © 2003 Examkrackers, Inc.

85

122.

123.

B.

Once the men had soldiered it would not be possible for them to return to slavery. WRONG: The argument does not imply this about the reasons for the North wanting to abolish slavery.

C.

The reasoning was probably not linked to the idea that men should not be slaves to one another. CORRECT: The argument does imply this about the reasons for the North wanting to abolish slavery. It is implied that the “differences” were the inherent superiority of whites over blacks. “The idea that ‘all men are created equal’ was based upon one’s definition of ‘men.’ [Implying that most white of the North and the South did not consider the blacks to be men.] To have invited an actual comparison of blacks and whites would have been met with incredulity, if not anger” (lines 25-28).

D.

Most of the white citizens of the North and the South still hated blacks. WRONG: The argument does not imply this about the reasons for the North wanting to abolish slavery. There is no real support for the idea of “hatred”. The passage is actually talking more about superiority and inferiority. For instance, a man may feel himself superior to a dog, yet not hate that dog.

Suppose that it can be proven that differences in human beings of the same gender are only skin deep. How would this information affect the claim that the “blacks were exceptional soldiers and had a better muscle structure than white soldiers” (lines 19-20)? A. It would support the claim. WRONG: This is not how the supposition/information would affect the claim. B.

It would refute the claim. WRONG: This is not how the supposition/information would affect the claim. Yes, it would refute the claim, but this answer provides no reason why, thus, it is not as good an answer as Answer D.

C.

It would refute the claim if it could be shown that Higginson was making a general reference. WRONG: This is not how the supposition/information would affect the claim. A generalization would allow for aberrations such as some white soldiers being more muscular than some black soldiers.

D.

It would refute the claim if it could be shown that Higginson was speaking about any and all soldiers. CORRECT: This is how the supposition/information would affect the claim. The supposition would clearly render Higginson’s statement incorrect if he meant that all black soldiers anywhere in the Army had a better muscle structure than any white soldier anywhere in the Army.

If the passage information is correct, what inference is justified by the fact that black soldiers were still going to war in segregated regiments, almost one hundred years after the period described? A. The black soldiers were still better organized than their white counterparts. WRONG: This inference is not justified by the “fact” in the question. The black soldiers were not segregated because they were better organized. B.

The attitudes and opinions of the white officers had changed very little. CORRECT: This inference is justified by the “fact” in the question. Why were the blacks segregated in the first place?

C.

The black soldiers were still being given secondhand clothing and equipment. WRONG: This inference is not justified by the “fact” in the question. This would more likely be a result of the segregation, not a reason or inference.

D.

Higginson’s dreams had not been fulfilled. WRONG: This inference is not justified by the “fact” in the question. There is no implication that Higginson dreamed of non-segregated units.

Passage VIII 124.

In lines 5-9, the author opines, “Readers may ask themselves whether, in the era in which George Sand wrote, it was possible for a woman writer to respond effectively to these questions while maintaining a traditional feminine role?” This question is most directly dealt with in the author’s discussion of: A. Bernard’s first meeting with Edmée. WRONG: This is not where the question is most directly dealth with.

Copyright © 2003 Examkrackers, Inc.

86

125.

126.

127.

B.

why Sand wrote Mauprat in the first place. WRONG: There is no information in the passage which would tell us “why” Sand wrote her novel.

C.

the changes which Edmée goes through. WRONG: This is not where the question is most directly dealth with.

D.

the advice of Virginia Woolf. CORRECT: This is where the question is most directly dealth with. The answer was, “yes”, but at a price. “Anticipating Virginia Woolf’s advice that ‘women...must cultivate...their masculine side,’ in order to transcend the limits of their sex in the act of writing.” (lines 13-18).

Which of the following statements best summarizes the main idea of the passage? A. Sand was tremendously influenced by other great female writers, such as Virginia Woolf. WRONG: This statement does not best summarize the main idea of the passage. B.

In order to write about power, goodness, and honor, Sand and her main character had to discard their traditional roles. CORRECT: This statement best summarizes the main idea of the passage. It is true that both Sand and Edmée did discard their traditional roles. It is equally true that, based upon the information in the second paragraph, it was necessary for Sand to discard her femininity in order to be taken seriously as a novelist.

C.

Sand’s life was very much mirrored by the character in her book, Edmée. WRONG: This statement does not best summarize the main idea of the passage. There is not enough information about Sand’s life to know if this is true or not.

D.

Though writing as a male, Sand did not really write like a male. WRONG: This statement does not best summarize the main idea of the passage. It may be true, but it is not as good an answer as Answer B.

The claim that Sand adopted her androgynous style to be considered an intellectual, necessitates which of the following conclusions? A. A woman of Sand’s day could never be considered an intellectual. WRONG: This conclusion is not necessary to the claim. “never” is an extreme word requiring strong support within the passage. It is conjecture to suppose that no women of that day could ever be considered intellectuals. The support for this conclusion is not in the passage. B.

Sand believed that a woman could not be considered an intellectual. CORRECT: This conclusion is necessary to the claim, based upon passage information.

C.

Androgynous writers were more likely to be considered intellectuals. WRONG: This conclusion is not necessary to the claim. This conclusion is not supported at all.

D.

Sand was probably already an intellectual, but felt that she wasn’t being accepted by women. WRONG: This conclusion is not necessary to the claim.

According to the passage, the title for Sand’s novel came from: A. George Sand’s real name. WRONG: This is not where the title for Sand’s novel came from. We are not provided Sand’s real name. B.

Edmée’s transformation. WRONG: This is not where the title for Sand’s novel came from.

C.

Bernard’s last name. CORRECT: This is where the title for Sand’s novel came from. At line 25 we learn that Bernard’s full name is “Bernard de Mauprat”.

D.

“angel of purity”. WRONG: This is not where the title for Sand’s novel came from.

Copyright © 2003 Examkrackers, Inc.

87

128.

129.

130.

Which of the following words would the author be most likely to use to describe a traditional woman from Sand’s period and place? A. passive, unconventional, poorly educated. WRONG: The author would not be most likely to use the word “unconventional” to describe a traditional wooman from Sand’s period and place. B.

androgynous, well-educated, bohemian. WRONG: The author would not be most likely to use the word “well-educated” to describe a traditional wooman from Sand’s period and place.

C.

listless, poorly educated, coquettish. WRONG: The author would not be most likely to use the word “listless” to describe a traditional wooman from Sand’s period and place.

D.

poorly educated, passive, beautiful. CORRECT: The author would be most likely to use these words to describe a traditional wooman from Sand’s period and place.

The passage discussion most clearly suggests the hypothesis that Bernard: A. became heartbroken over his loss of Edmée. WRONG: This is not most clearly suggesed. B.

came to care and respect Edmée for who she was, more than what she looked like. CORRECT: This is most clearly suggesed. Lines 47-57 show this transformation.

C.

was able actually a homosexual. WRONG: This is not most clearly suggesed.

D.

no longer felt that Edmée was physically beautiful or charming. WRONG: This is not most clearly suggesed. Simply because he ignored her physical beauty does not indicate that he didn’t find her still beautiful. Additionally, it is not at all clear that he didn’t still find her charming.

What is the intended relevance of the comment that, “She understood that the woman writer must attempt to transcend gender in order to escape a female ghetto of the imagination which is not of her own making, and to which men have relegated her” (lines 15-18) to the rest of the passage? A. To explain why Sand was essentially forced to become androgynous in order to write a serious novel CORRECT: This is the intended relevance to the rest of the passage. B.

To indicate that not only Virginia Woolf, but other critics also agreed with what Sand was saying and doing WRONG: This is not the intended relevance to the rest of the passage.

C.

To express the sentiments of women in general towards a society which had repressed them WRONG: This is not the intended relevance to the rest of the passage.

D.

To provide the basis for further discussion regarding the similarity between Sand and Edmée WRONG: This is not the intended relevance to the rest of the passage.

Passage IX 131.

Suppose the author of the passage is criticized for not seeing the similarities between the wrong-headed exploits in The Girl Who Lived at the Ritz and Wind in the Willows. The author of the passage would be most likely to respond to this criticism by: A. suggesting that, unlike Eloise, Mr. Toad is usually faced with the consequences of his poorly made decisions. CORRECT: The author would be most likely to respond to criticism in this way. The author complains that the popular Eloise sets a poor example in getting into trouble which her rich mother always gets her out of. Though the author doesn’t compare the two, Mr. Toad apparently suffers somewhat from his escapes. This answer would allow the author to support his arguments and passage rather than admitting he was wrong. B.

proposing that the difference between the two lies in comparing humans to animals, which is not possible. WRONG: The author would not be most likely to respond to criticism in this way. There is no support for this in the passage.

Copyright © 2003 Examkrackers, Inc.

88

132.

133.

134.

135.

C.

asserting that a toad wrecking an automobile is nothing compared to a little girl ruining a wedding party. WRONG: The author would not be most likely to respond to criticism in this way. This is pure conjecture with no basis.

D.

explaining that one should not take a children’s book too seriously. WRONG: The author would not be most likely to respond to criticism in this way.

The author’s comparison of Miss Piggy with “the Miss Piggy-prototype Snork Maiden, who considers herself very comely” (lines 33-35) indicates that: A. the Snork Maiden actually considers herself rather dowdy. WRONG: This is not indicated by the author’s comparison. B.

the Snork Maiden existed before Miss Piggy. CORRECT: This is indicated by the author’s comparison. If the Snork Maid is the protype, she came first.

C.

Miss Piggy is probably not a pig at all. WRONG: This is not indicated by the author’s comparison.

D.

both of the characters are ‘women’ dressed as men. WRONG: This is not indicated by the author’s comparison.

If the author of the passage admired the Curious George children’s series, where the little monkey George is constantly getting into mischief before being admonished and then allowed to repair his damage, this admiration would be most discrepant with the passage assertion that: A. The Girl Who Lived at the Ritz has been declared a “children’s treasure”. WRONG: The author’s admiration would not be most discrepant with this assertion. The author admits to the book being a treasure without agreeing with this appelation. B.

As for darling Eloise, she seems to be a spoiled and irritating brat. WRONG: The author’s admiration would not be most discrepant with this assertion. It might not be discrepant at all if one considers that George is apparently required to “repair his damage”.

C.

Despite its popularity, the The Girl Who Lived at the Ritz series sets a bad example for young readers. CORRECT: The author’s admiration would be most discrepant with this assertion. Though the idea is somewhat muddied with the author’s admiration for Mr. Toad, the point is made in the passage that the author thinks that characters in the children’s books should set a good example for young readers.

D.

The immensely popular Wind in the Willows series is a true work of children’s literature. WRONG: The author’s admiration would not be most discrepant with this assertion.

In the context of the passage, the term timeless (lines 1-7) refers primarily to: A. our not knowing precisely when the books were published. WRONG: This is not what the word primarily refers to. B.

books that have been, and will be enjoyed by generations to come. CORRECT: This is what the word primarily refers to. This answer is clearly the author’s meaning and, unlike Answer C, requires no interpretation.

C.

books that will never grow old. WRONG: This is not what the word primarily refers to. This answer is not as specific as Answer B. It requires interpretation of “never grow old”. If one takes this answer literally, it is quite incorrect.

D.

only the Dr. Seuss books. WRONG: This is clearly not what the word primarily refers to.

If the author were to characterize the Moominvalley series by Tove Jannson, he would most likely use the words: A. totally unknown; beautiful illustrations; reasonably priced; highly recommended. WRONG: These are not the words the author would be most likely to use. The books are “largely unknown” (lines 24-25 which differs completely from “totally unknown”.

Copyright © 2003 Examkrackers, Inc.

89

136.

137.

B.

priceless; overrated; wondrous; unceasing misery. WRONG: These are not the words the author would be most likely to use. The author would not call the books “overrated”.

C.

delightful characters; over-priced; largely unknown; timeless. WRONG: These are not the words the author would be most likely to use. The books are described as “reasonably priced” (line 43).

D.

timeless; delightful characters; reasonably priced; not well known. CORRECT: These are the words the author would be most likely to use. In fact, these words, or words similar to them are all used in the third paragraph to describe the series.

The passage suggests that its author would probably disagree with which of the following statements? A. The popularity of a children’s book is based upon the example set by its main character. CORRECT: The author would most probably disagree with this statement. Based the fact that The Girl Who Lived at the Rizt has been declared a children’s treasure and is thus, presumably popular, yet the author thinks that the main character does set a bad example, we can come to the conclusion that the author would not agree with this statement. B.

A child’s age should be taken into account when deciding upon the ideal book. WRONG: The author would not most probably disagree with this statement. We really don’t know if the author would disagree or not with this statement.

C.

There are only a few children’s books in which the characters set a poor example. WRONG: The author would not most probably disagree with this statement. We really don’t know if the author would disagree or not with this statement.

D.

Stories with humans as characters can be just as popular those using animals. WRONG: The author would not most probably disagree with this statement. We really don’t know if the author would disagree or not with this statement.

The author’s reasoning about what constitutes “highly recommended for children” (line 44), could most reasonably be extended to questions about: A. action figurines. WRONG: The author’s reasoning could not be most reasonably extended to questions about this subject. An action figure is an inanimate object. Don’t read into the question or answer. B.

toy guns. WRONG: The author’s reasoning could not be most reasonably extended to questions about this subject. Toy guns are inanimate.

C.

television programming. CORRECT: The author’s reasoning could be most reasonably extended to questions about this subject. The author is concerned with examples set for young children. This requires some sort of active medium.

D.

snack foods. WRONG: The author’s reasoning could not be most reasonably extended to questions about this subject.

Copyright © 2003 Examkrackers, Inc.

90

Answers & Explanations Biological Sciences Questions 138-214

91

Passage I (Questions 138-144) 138. C is correct. According to the information presented in the passage (last paragraph), IGF-1 is an autocrine/paracrine mediator, which by definition causes a local effect on target tissue. Therefore, because IGF-I is synthesized in the liver it must also act on liver hepatocyte receptors. Hepatocytes are liver cells involved in functioning metabolism. Astrocytes are nervous system cells that support and provide nutrients to neurons. Kidney parenchymal cells represent renal structural components not involved in plasma filtration. A kidney nephron is not considered to be a parenchymal cell.

portant in nature? Viral particles must attach to specific cell surface receptors in order to gain entry into host cells. This most certainly is an important biological interaction. Table 1 informs that T-cell receptors bind Interleukin-2 (IL-2). How about IGF-1? According to the passage, IGF-1 is structurally similar to insulin. Structurally similar compounds often share receptors; therefore it can be assumed that IGF-1 can bind insulin receptors. 144. C is correct. Cancer is defined as uncontrolled, improper growth that fails to respond to normal cell cycle controls. According to the passage, IGF-1 causes a switch from a “non-mitogenic” to a “mitogenic” mode. The key to realize is that IGF-1 is a mitogen not a mutagen. Mutagens are toxins that cause DNA damage and may lead to development of neoplastic (cancerous) tissue. An over-expression of IGF-1 receptors will result in increased sensitivity to circulating IGF. This will lead to excessive fibroblast cell growth, which may result in cancer. Fibroblasts are structural components of breast tissue.

139. C is correct. Oxytocin is a hormone produced by the hypothalamus, stored in the posterior pituitary, and functions to 1) stimulate mammary gland milk ejection and 2) initiates uterine smooth muscle contraction to facilitate parturition or birth. Prolactin induces growth of the mammary gland and stimulates milk production. Pitocin must be a synthetic version of oxytocin. Testosterone is produced in the testes by Leydig cells and is imperative for proper male development. Progesterone is important during the menstrual cycle and pregnancy.

Passage II (Questions 145-150) 145. D is correct. Just as there are SN2 and SN1 reactions, there are two types of elimination reactions: E1 and the E2. The difference between the two is that an E2 reaction proceeds by second-order kinetics and involves a single step; whereas an E1 reaction proceeds by firstorder kinetics and is a two-step reaction. Any product of an elimination reaction invariably undergoes the formation of a “new” double bond. Compound 5 to lanosterol conversion is a classic example of an E1 reaction.

140. B is correct. According to the passage, the autocrine system relies on a “vast array of low molecular weight proteins”. Rough endoplasmic reticulum (ER) is an organelle that houses many ribosomes, which serve as the site for protein synthesis. Protein synthesis also takes place in the cytoplasm, where “free”, non-ER bound ribosomes are present. Smooth ER is responsible for cell detoxification and lipid synthesis (i.e. steroids). It is a microscopic liver at the cellular level. Mitochondrion is the site of cellular respiration and is considered to be the “power house” of the cell. Lysosomes are double-membrane structures that carry out hydrolysis of ingested compounds.

146. B is correct. Steroid hormones act as a messenger system providing communication between cells of a multicellular organism which require such communication for survival. Prokaryotes are unicellular and do not require this type of intercellular communication for their survival. Independent survival is a basic difference between cells of unicellular organisms and those of multicellular organisms.

141. B is correct. According to the passage, growth factors (i.e. IGF) act as mitogens by stimulating cellular proliferation and survival, therefore inhibiting apoptosis, which causes cell death. Normally, apoptotic programmed cell death occurs during 1) development (i.e. finger formation in a fetus) and 2) cancer cell elimination. IGF levels are carefully monitored: being mitogens by nature, these compounds have the potential to initiate a cancerous or neoplastic event.

147. B is correct. According to the passage, the squalene epoxide is acid catalyzed into a multi-step cyclization cascade that yields lanosterol. AlF3 is an electron pair acceptor. A chemical species that accepts a pair of electrons is a Lewis acid. Thus, the correct answer is B. NaOH and NH3 are both bases.

142. D is correct. According to the passage, paracrine signals are released by cells into the extracellular space and act only locally. ACTH and thyroid hormones are endocrine mediators: acting on organs a great distance away from the secretory site. Testes are organs, involved in spermatogenesis and testosterone production. The best way to answer this type of question is through the process of elimination. Prostaglandins are paracrine mediators involved in pain and inflammation. Aspirin prevents prostaglandin formation by inhibiting COX enzymes and as a result inhibits headache pain.

148. D is correct. Carbonyl compounds (i.e. aldehydes) typically display sharp, narrow IR bands in the 1650 – 1780 cm–1 region. Compound 3 is an alcohol because of the hydroxyl group. Alcohols display broad IR bands in the 3200 – 3500 cm–1 region. The correct answer is D. 149. C is correct. Lipoproteins are categorized according to protein density. According to Table 1, as percentage of

143. D is correct. So what cell surface interactions are imCopyright © 2003 Examkrackers, Inc.

92

protein increases lipid concentration decreases. In comparison to other lipoproteins, high-density lipoprotein (HDL) is the “good cholesterol” because of its lowest lipid to protein ratio.

154. C is correct. You should know that calmodulin and cAMP are both second messengers, and that’s all there is to this question. You may also see that they both activate other proteins in the cytoplasm.

150. C is correct. Answer choice D is cholesterol, not an intermediate compound. Compound 3 and 5 are both fourringed structures. The reaction intermediate as a result must also be a four-ringer structure. The correct answer must be C.

155. C is correct. This question sound complicated. One must realize however, that all is being asked is which hormone is not produced by the hypothalamus-pituitary axis. The hypothalamus-pituitary endocrine axis is responsible for overall body homeostasis. The hypothalamus regulates all secretions of the pituitary gland. All six anterior pituitary hormone products—TSH, Prolactin, FSH, LH, ACTH, and Growth Hormone—are of peptide nature and, according to the question stem, are cAMP dependent. Peptide hormones are hydrophilic—they cannot cross the hydrophobic phospholipid bilayer—and therefore bind receptors on the surface of target cell membranes, which initiates an intracellular second messenger (i.e. cAMP) cascade to modify RNA transcription rates. Cholesterol derived steroid hormones (i.e. aldosterone) bind nuclear receptors directly and therefore do not depend on second messengers.

Passage III (Questions 151-157) 151. D is correct. According to the passage, intrinsic activity (maximum being 1.0) is the efficacy of the transduction process between receptor binding and drug response. In order to have an intrinsic activity value, drug-receptor binding must occur. Not all receptors are enzymes and intrinsic activity is not designed to measure abnormal catalytic states. Drugs with intrinsic activity of less than 1.0 are partial agonists: meaning that they bind to a receptor but fail to stimulate a maximum reaction. A drug with intrinsic activity of 0.0 binds to a receptor but fails to stimulate. It is therefore best to classify this compound as an inhibitor: the receptor is blocked, yet no activity occurs.

ADH is released from the posterior pituitary and stimulates aquaporin channel production, which increases water reabsorption at the nephron collecting duct. This leads to increased blood pressure. The correct answer is C.

152. D is correct. Figure 2 shows 5 G-proteins activated for each signaling molecule, five adenylyl cyclases activated for each G-protein, five cAMPs for each adenylyl cyclase, five kinases for each cAMP, 5 enzymes for each kinase, and 5 enzyme products for each enzyme. This would lead to 55 enzyme products for each signaling molecule.

156. B is correct. Figure 1 displays four unique transmembrane signaling mechanisms. The question asks which pair, out of the four receptors presented, is most similar. The key point to realize is that receptor DD is a G protein. Receptor AA is utilized by a lipid-soluble ligand (i.e. cortisol) to diffuse across the membrane to stimulate its intracellular receptor. Transmembrane receptor BB has an extracellular domain that binds ligands, which activates enzymatic activity of intracellular receptor domain. Therefore, receptor BB, being a transmembrane molecule with enzymatic activity is most similar to a G protein. Receptor CC is attached to a ligand-gated ion channel.

153. A is correct. There seems to be a disconnect between the question and the answer choices. The question asks for an explanation for ‘duration’, and the answer choices provide information on ‘how strong’. Nevertheless, there is a clear best answer based on basic science knowledge, the amplification discussed in the passage, and common sense. It doesn’t coincide with our science knowledge that all receptors or exactly half the receptors must be occupied to create a response. We know that chemical reactions occur due to collisions, and that all reactants don’t collide with all products in a single moment. Also, the passage shows G-proteins activated by the binding of one receptor and signal molecule. There is no mechanism shown to interrupt this activation until other receptors are bound. In reality, cell surface receptors occupy less than 1-percent of total cell surface area. Nevertheless, to occupy all surface receptors is nearly impossible and requires an extremely high ligand concentration. Nature overcomes this by amplifying incoming extracellular signals via a multi-step signal cascade. One example is presented in Figure 2. Amplification allows for minute quantities of messenger to initiate large cellular events such as movement of limbs or protein synthesis. Copyright © 2003 Examkrackers, Inc.

157. B is correct. Growth factor is a signaling molecule that binds to the receptor, as per the question. From the passage and Figures 1 and 2, receptors are on the membrane.

93

Passage IV (Questions 158-164) 158. C is correct. According to the passage, a carrier of a balanced reciprocal translocation has an increased chance of having abnormal progeny. Thus, normal progeny is possible. As with any genetic disease, a phenotypically normal carrier of a reciprocal translocation can have: 1) phenotypically normal progeny; 2) an offspring that is a carrier of the balanced translocation; 3) progeny with various unbalanced arrangements (i.e. monosomies). A balanced genome that contains 45 chromosomes is seen in Robertsonian, not reciprocal, translocations.

165. C is correct. Cell membrane is a hydrophobic structure composed of phospholipids and proteins. Integral proteins, which span the entire thickness of the membrane, function as carriers and must form hydrophobic interactions with surrounding hydrophobic fatty acids. Phospholipid phosphate groups (negatively charged) are hydrophilic and interact well with polar compounds like water. 166. C is correct. Disaccharide digestion and absorption occurs at the microvilli (also known as a “brush border”) of the small intestine. Incorporated into the brush border are enzymes that complete digestion: maltase—hydrolyzes maltose into two molecules of glucose; sucrase —hydrolyzes sucrose into glucose and a fructose; and lactase - hydrolyzes lactose into glucose and a galactose. Carbohydrate polymer (i.e. starch) digestion begins in the mouth. However, tiny disaccharides are not digested until chyme (food mass leaving the stomach) enters the small intestine. Large intestine (colon) is mostly responsible for water reuptake.

159. C is correct. The main thing to realize is that the question stem discusses single-gene disorders. A single gene or allele mutation must be dominant in nature to be seen phenotypically. Autosomal recessive genotypes show penetrance only when both genes are either damaged or deleted. Neurofibromatosis is an example of an autosomal dominant condition that leaves individuals susceptible to cancer. 160. B is correct. According to the passage, Robertsonian translocation is the centromeric fusion of two non-homologous acrocentric chromosomes. Answer choices C and D are therefore incorrect. It can be inferred from the passage that acrocentric chromosomes possess very long q-arms and a tiny p-arms. No information was provided regarding chromosome stability or break frequency. However, chromosome breakage is associated with reciprocal translocation.

167. B is correct. Disruption of hydrogen bonds, which weakly hold two complementary DNA strands, results in denaturation of DNA. Phosphodiester bonds link individual nucleotides together producing single stranded DNA. Breaking these bonds will completely destroy DNA, but not denature it. Denaturation implies that DNA will reform if negative forces are removed. 168. A is correct. Gulose is a pyranose (six-membered ring) and is a beta-anomer (C1 hydroxyl group is in the equatorial position).

161. A is correct. Disjunction is the separation of sister chromatids during anaphase. ‘Primary’ refers to the firs anaphase. 162. B is correct. According to the question stem, abdominal infections lead to scar tissue formation, which can sometimes form a complete barrier between the ovary and the fallopian tube and cause infertility. This woman will still be able to ovulate. However, her ova would never reach the uterus. 163. C is correct. Tissue is a group of cells that usually has a common embryonic origin and functions together to carry out a specific task. Mitosis is an ongoing process designed to grow, repair (scar tissue formation) and replace tissue cells when necessary. Transduction is a virally induced genetic information transfer from one cell to another. 164. C is correct. The white chromosome is chromosome 21 from Figure 1. We need three of these for trisomy 21. Choice C is the only one. The question also says that the offspring will have 46 chromosomes, this should narrow the choices to B or C. Finally, only choice C has the chromosomal mutation resulting from a Robertsonian translocation as shown in Figure 1. The chromosomes in the other choices are all normal. Free Standing (Questions 165-168) Copyright © 2003 Examkrackers, Inc.

94

Passage V (Questions 169-173) 169. A is correct. After carefully examining Figure 1, it becomes apparent that morphine is a penta-cyclic compound: meaning its structure contains five rings. Naturally occurring plant source amines, like morphine, are referred to as alkaloids. However, that answer choice is not an option. Morphine also contains two hydroxyl groups, which would classify it as an alcohol. Answer choice A is correct.

176. B is correct. Cells will adapt to changing environments by undergoing processes like atrophy (loss of organ size), hypertrophy (swelling),and hyperplasia (increase in cell number). However once a cellular threshold is reached, cells will begin to undergo necrosis, meaning death. 177. B is correct. Interstitial cells of Leydig produce cholesterol-derived testosterone. Cholesterol derived products are synthesized by the smooth ER and are secreted by a well-developed golgi apparatus. These processes are ATP dependent and require a high cellular concentration of mitochondria.

170. B is correct. By definition, a secondary alcohol is attached to a secondary carbon. A secondary carbon is bonded to two other carbons. The correct answer is B. Answer choice A is a primary alcohol. Answer choice C is a tertiary alcohol. Answer choice D is an ester. 171. D is correct. Electrophiles are positively species and/or Lewis acids: Lewis acids are pair acceptors. Nucleophiles are negatively species and/or Lewis bases. Lewis bases are pair donors.

Passage VII (Questions 178-183) 178. C is correct. Detoxification takes place mainly in the liver.

charged electron charged electron

179. D is correct. According to the passage, biotransformation reactions convert lipid-soluble parent compounds, which can easily diffuse into cells, into polar metabolites with greater dielectric constants. Increased polarity increases water solubility and decreases lipid solubility. The altered chemical properties decrease metabolite permeability across biological membranes and promote its excretion. Biotransformation may also exclude metabolites from their biological site of action. However, the biotransformed metabolites are not necessarily rendered completely inactive. In fact, some metabolites of drugs are actually more active than their parent compound.

172. C is correct. From the passage, at least some of the activity of morphine is due to the methylated nirogen, a property shared by oxymorphone. Thus, oxymorphone is likely to have analgesic effects. Choice A is incorrect. The passage states that morphine and codeine are the only two naturally occuring plant products that have analgesic properties. Choice B is incorrect. A comparison of the structures of morphine and oxymorphone would indicate that oxymorphone is a derivative of morphine. Choice C is correct. There is nothing to indicate that choice D is true.

180. C is correct. Three atoms – oxygen, nitrogen, and fluorine - with attached hydrogen molecules are capable of hydrogen bonding. For example, H—F, COOH and NH2 can hydrogen bond with water and other polar compounds. C—H bond is not considered to be water-soluble and does not hydrogen bond with polar compounds

173. D is correct. Only choice D is a N-demethylated alkaloid (demethylated oxycodone) as described in the passage as an inactive compound. Choice A is Heroin, choice B is oxycodone, choice C is codeine.

181. C is correct. According to the passage, P450 dependent oxidation is the most important Phase I reaction. When examining Figure 1, 5 percent of acetaminophen is converted to acetaminophen N-hydroxyl by the mixed function oxidase.

Passage VI (Questions 174-177) 174. C is correct. Testosterone inhibits GnRH, LH, and FSH production reducing the activity of the Sertoli cells in spermatogenesis and reducing the production of testosterone by the cells of Leydig. The extra testosterone will increase secondary sex characteristics such as increased muscle mass.

182. B is correct. The liver is the most important organ of detoxification in terms of concentration of toxin-metabolizing enzymes and size. The endoplasmic reticulum contains mixed function oxidases, which use the cytochrome P450 for the addition of oxygen to substrates. Mitochondria are capable of oxidative deamination via monoamine oxidase. Cytosol contains non-organelle associated enzymes such as alcohol dehydrogenase.

175. A is correct. Testosterone, male sex hormone, is produced by Leydig cells, which according to the passage possess LH receptors. Thus, LH controls testosterone production in males. FSH is responsible for sperm maturation and binds Sertoli cells.

183. A is correct. According to the passage, endoplasmic reticulum contains numerous mixed function oxidases, which use cytochrome P450 for the addition of oxygen to substrates. So, P450 produces highly oxidized metabolites. The question stem asks the reader to select an oxidation reaction where the metabolite contains an increased molar concentration of oxygen than the parCopyright © 2003 Examkrackers, Inc.

95

ent. Answer choices B, C, and D are reduction reactions. In reduction reactions molar concentration of oxygen stays the same as hydrogen molar concentration increases.

blood pressure in the left ventricle during contraction. The diastolic or bottom number represents blood pressure in the left ventricle while it is being filled. Free Standing (Questions 189-195) 189. C is correct. Atom’s valence electron number determines how many bonds that atom can create. Molecule A has 2 carbons – 4 valence electrons each; 6 hydrogens – 1 valence electron each; and 1 sulfur – 6 valence electrons: that brings the grand total to 20 valence electrons and 20 possible bonds.

Passage VIII (Questions 184-188) 184. D is correct. As stated in the passage, edema is increased fluid in the intracellular or interstitial space. As blood flows through capillaries, some of it is filtered through capillary walls into neighboring tissue, which is known as the interstitial space. Increased hydrostatic pressure within the capillary forces more fluid from inside the capillary through the capillary pores into the interstitium. Decreased plasma osmotic pressure within the capillaries results in weaker “pull” of fluid back into the capillaries from the interstitium, which also results in increased edema. Increasing the permeability of the capillary walls allows more fluid to move from within the capillary into the interstitial space, resulting in increased edema. Only the increased flow of lymph would reduce the amount of edema since the lymphatic system drains the interstitial space.

190. C is correct. Bacterial pili is involved in a biological process that represents the closest thing to sexual intercourse at the cellular level. Conjugation is the direct acquisition of DNA from another bacterium via the sex pili. Transformation involves the acquisition of DNA “left” in the environment (simple “see and grab”). The important thing to realize is that genetic material must be of appropriate size and polarity to be picked up and utilized by the cell. Transduction is defined as DNA acquisition via a bacteriophage intermediate. As viruses jump from host to host, they drag foreign DNA with them. These 3 mechanisms allow bacteria to undergo recombination and increase genetic variability. Translocation is the rejoining of broken chromosome ends.

185. B is correct. According to figure 1, only two factors directly affect blood pressure: cardiac output and peripheral resistance. All other factors listed in the figure do affect blood pressure, but do so indirectly. 186. B is correct. The heart and kidneys normally control blood pressure. The heart regulates the amount of blood that’s being pumped out per unit of time (cardiac output). The kidneys regulate blood pressure by increasing or decreasing renin in response to changing circulating blood volume. Blood pressure is highest at the aorta, since it is where the blood goes immediately after having been pumped out of the left ventricle. The pressure continues to drop as blood circulates through the body, ultimately returning to the right atrium through the inferior vena cava at lowest pressure. Lymph flow is mostly unrelated to blood pressure.

191. B is correct. CH2 CH2 CH2 Doesn't Resonate

Resonance involves electron delocalization, which increases overall compound stability. Answer choice B displays a change in the actual position of atoms: this does not occur in resonating structures.

187. C is correct. Left to its own devices, blood would tend to pool in the lower extremities when a person moves from the lying down to the standing position. This would cause a massive decrease in overall blood pressure, since most of the blood is pooled at the feet and very little is actually circulating. This dizziness is what is known as orthostatic hypotension. To prevent it from happening, the body normally offsets the decrease in blood pressure by increasing one or more of the factors that directly regulate blood pressure. We see in figure one that those two factors are cardiac output and peripheral resistance. Peripheral resistance is increased via the sympathetic system.

192. B is correct. RNA (single stranded, with adenine binding uracil, not thymine) is produced in the nucleus by the process of transcription. DNA serves as a template for RNA polymerase to synthesize mRNA - determines amino acid sequence in the protein being produced. Thus, answer choice D is eliminated. mRNA migrates into the cytoplasm where ribosome/rRNA complex rRNA is produced in the nucleolus and is necessary for proper ribosome function - binds mRNA to search for the start codon (AUG). During this stage, tRNA/amino acid complexes bind to the appropriate mRNA codon. tRNA carries amino acids to the ribosomal site for protein synthesis. Non-coding introns are subtracted from the coding exons in order to transcribe mature mRNA.

188. A is correct. A blood pressure reading compares the pressure within the heart at its peak (during contraction), to its pressure at its lowest (while the ventricles are relaxed and filling). The systolic or top number represents Copyright © 2003 Examkrackers, Inc.

CH2

193. A is correct. Negative feedback is seen when the output

96

of a pathway, in this case FSH, becomes elevated and inhibits further FSH production by the anterior pituitary or further stimulation by the hypothalamus. All hormones produced by the anterior pituitary, with the exception of prolactin, are controlled via the negative feedback mechanism. Conversely, when seminiferous tubules fail to produce sperm because of low FSH, the secretion of FSH by the anterior pituitary increases. Positive feedback occurs when pathway product, for instance prolactin, stimulates further prolactin synthesis. Competitive inhibition occurs in enzyme kinetics.

199. D is correct. According to Table 1, HCl secretion mechanism is as follows: gastrin is released and stimulates parietal cells to release HCl. HCl is needed for pepsinogen activation. In gastric ulcer disease, gastrin levels are increased not decreased. Answer choice A must be incorrect. Answer choice C is a possibility, however, gastrin receptor down regulation does not occur in Zollinger-Ellison syndrome, in which gastrin levels are even higher. Therefore, the correct answer must be D. 200. C is correct. HCl and intrinsic factor are secreted by parietal cells. Chief cell produce pepsinogen, while mucous cells secrete basic mucus. The stomach has no such thing as an islet cell.

Passage IX (Questions 194-197) 194. A is correct. Carbon 1 is the carbonyl of the aldehyde. The hydroxyl group on carbon 5 attacks carbon 1 and the carbonyl oxygen may point up or down.

201. C is correct. According to Table 1, Zollinger-Ellison syndrome increases gastrin secretion, which stimulates H+ release into the lumen of the stomach. Low stomach pH breaks down the mucosal barrier, which cannot divide rapidly enough to provide adequate protection.

195. D is correct. Substitution reaction rates are governed by the quality of the leaving group. I- is the best halide leaving group. Iodine forms the weakest bond with the parent molecule and is therefore very efficient at rapid dissociation when the reaction is taking place.

202. A is correct. The parasympathetic stimulation results in increased activity of the gastrointestinal tract. This includes increased production of saliva and increased acid secretion. Therefore, if the parasympathetic stimulation is inhibited, acid secretion would decrease. The autonomic nervous system relies on acetylcholine in its parasympathetic arm and norepinephrine in its sympathetic division. Stomach chief cells produce pepsinogen and their removal will not cure ulceritis. GI parenchyma is structural tissue that surrounds the digestive tract and the stomach.

Details regarding SN2 substitution reaction: • • • • • • •

one step reaction follows second order reaction kinetics (rate = k [S][Nucleophile]) proceeds through a transition state prefers primary carbons prefers aprotic solvent produces optically active product causes an inversion of stereochemistry

Passage IX (Questions 203-208) 203. B is correct. We learn from the passage that uric acid is a common solution to the potential problems posed by trying to excrete nitrogenous waste in a shelled egg. Of the choices given, only birds lay shelled eggs.

196. D is correct. 2R, 3R dichloropentane and 2S, 3S dichloropentane are enantiomers. By definition, enantiomers have an equal in magnitude and opposite in sign specific rotation. Furthermore, enantiomers share identical chemical and physical properties, which make it very difficult and costly to separate them out.

204. D is correct. The definition of “vestigial” (i.e., left over from the past) gives this one away. A is out because such a thing doesn’t happen in nature, at least not the growing of bones. B is tricky: it indicates that the snakes evolved into limbed reptiles. But again, “vestigial” argues against this answer. C is a false statement.

197. C is correct. According to the passage, intramolecular nucleophilic addition can take place between a hydroxyl and a carbonyl group of the same molecule, resulting in the formation of a cyclic hemiacetal. Answer choice C is the only compound that fails to meet the requirement because it lacks a carbonyl group. Answer choice A, B and D compounds can all for an intramolecular hemiacetal.

205. A is correct. We learned in the passage that the skin could be used as a gas-exchange surface, and the skin is fed by the systemic circulatory system. B is untrue. C is false because higher blood pressure normally allows for greater activity, not lower blood pressure.

Passage X (Questions 198-202) 198. C is correct. According to the passage, H. pylori’s urease converts urea into ammonia, which is a base. Moreover, ammonia allows bacteria to survive in a highly acidic environment by buffering stomach H+. The human body utilizes bicarbonate (HCO3-) to buffer most body fluids. Furthermore, bicarbonate is the only base listed.

Copyright © 2003 Examkrackers, Inc.

206. A is correct. Any chordate possesses a notochord. Any vertebrate is also a chordate, so if we know that an organism possessed a vertebral column, it must also at some point possess a notochord. B is out because the notochord does not become the spinal cord in reptiles. As for choice C, the statement is correct—some species do possess notochords and do not have vertebral columns— but such a fact does nothing to argue against the stu-

97

dent’s conclusion. While a species can possess a notochord without possessing a vertebral column, a species cannot possess a vertebral column without possessing a notochord at some point during development. For choice D, the statement is again true, but it does not impact the student’s conclusion.

211. A is correct. The reaction mechanism is of an addition reaction because the bonding of the hydroxyl group failed to displace any of the previously attached substituents. The π bond is weaker than its sigma counterpart. A weaker bond requires less energy to break it. 212. D is correct.

207. C is correct. The passage says that uric acid replaces urea. You should know that nitrogen is an element in the waste product urea, and therefore is also found in urea. 208. A is correct. r strategists reproduce far more offspring than survive to reproduce. Free Standing (Questions 209-214) 209. A is correct. According to the question stem, cytomegalovirus genome is quite homologous with eukaryotic DNA. This implies that viral DNA would be easily integrated into eukaryotic genome and/or had been obtained from mammalian cells. Thus, answer choice A displays the best fit with the concept presented in the question stem. Answer choice B, herpes virus is a eukaryotic branch mutation, is false. Choice C is too extreme in stating, “most eukaryotic promoter/enhancer regions are identical to those of cytomegalovirus”. Finally, answer choice D is incorrect because evidence to support the statement is lacking.

Xn

Y

X

XXn

XY

X

XXn

XY

Key: Xn = Diseased We must first assume that disease Z is x-linked since none of the boys (who received only a y-gene from their father) have the disease. To determine whether the disease is dominant or recessive we must examine the daughters. According to the question, all of the daughters have the disease. Since we know that the x gene the girls received from their mother is normal, the disease must be on the x-gene received from their father. It must also be a dominant for them to express the disease with only one diseased gene.

210. B is correct. Glycogen and mucopolysaccharides are polymers used for long-term glucose storage. These will be packed into vesicles until needed for metabolism. Glucose is not found in the nucleus or the cell membrane.

213. B is correct. H

OCH2CONH

1

H

CH3

S

CH3

2

N O

3

H COOH

Penicillin V The presence of a stereogenic center defines a compound as chiral. The most common cause of chirality in organic molecules is the presence of a tetrahedral carbon atom bonded to four different substituents. Penicillin V has 3 stereogenic centers. 214. C is correct. Acrosome is a vesicle located on the head of the sperm that contains the enzyme hyaluronidase, which allows the breakdown of the protective barrier surrounding the ovum. The acrosome is equivalent to the lysosome of a “typical” cell. Sperm head contains the genetic material, DNA, and is equivalent to a cell nucleus. The mid-piece contains mitochondria, which produce energy in the form of ATP, which moves the tail and propels the spermatozoa forward. The tail is a microtubular structure that helps to propel sperm up the female reproductive tract.

Copyright © 2003 Examkrackers, Inc.

98